Đến nội dung

Sk8ter-boi nội dung

Có 427 mục bởi Sk8ter-boi (Tìm giới hạn từ 25-04-2020)



Sắp theo                Sắp xếp  

#194419 Giải thế nào đây?

Đã gửi bởi Sk8ter-boi on 06-12-2008 - 12:38 trong Phương trình - Hệ phương trình - Bất phương trình

bài này nếu là x>= 0 là ổn r?#8220;i :D ,
x<0
$\dfrac{1}{x^2}=2^{-x} $
$\dfrac{-1}{x} = 2^{\dfrac{-x}{2}} $
$\dfrac{-1}{x} = e^{-\dfrac{ln 2}{2}x} $
$\dfrac{-x\dfrac{ln2}{2}}{-x} = (-\dfrac{ln2}{2} x ) e^{-\dfrac{ln 2}{2}x} $
$W(\dfrac{ln2}{2} ) = -\dfrac{ln2}{2} x $
$x=-\dfrac{2}{ln2}W(\dfrac{ln2}{2} ) $
W là hàm lambert W , lambert W -function



#193513 giai ho em bai nay voi

Đã gửi bởi Sk8ter-boi on 16-11-2008 - 17:49 trong Các bài toán Đại số khác

BDDT Schur :
Với mọi số thực không âm a,b,c :
$ a^3 + b^3 + c^3 + 3abc \ge ab(a + b) + bc(b + c) + ac(c + a) $

BDT Schur :
$\sigma a^r(a-b)(a-c) \geq 0$
bdt bạn nêu trên là với r=1 .
bạn có thể xem chi tiết trong các sách BDT hoặc xem tại link sau BDT SCHUR



#193512 Giới hạn

Đã gửi bởi Sk8ter-boi on 16-11-2008 - 17:44 trong Dãy số - Giới hạn

Làm dùm mình bài toán này luôn

$(u_n): u_n=1+\dfrac{1}{2^k}+\dfrac{1}{3^k}+...+\dfrac{1}{n^k}$
với $k$ cố định, $k>1$
Tìm $\lim_{n\to \infty} u_n$.


bạn lên mạng search "zeta function" là có đấy :perp
hoặc bạn thử 2 links này xem :vdots :sum:limits_{i=1}^{n}
http://en.wikipedia....i/Zeta_constant
http://en.wikipedia....n_zeta_function



#193292 tìm a

Đã gửi bởi Sk8ter-boi on 05-11-2008 - 20:42 trong Các bài toán Đại số khác

k bít cái này cho vào box cấp 3 có đúng k<hay THCS _ __"> ^.^ vì mình dùng cấp 3 để giải _ __"
tìm a để
$x^2+x+6 | x^n+x+a$ với n tự nhiên



#193219 $\int\limits_{0}^{\dfrac{ \pi}{2} } sin^nx dx$

Đã gửi bởi Sk8ter-boi on 03-11-2008 - 22:04 trong Tích phân - Nguyên hàm

tính $u_n$ : $\int\limits_{0}^{\dfrac{ \pi}{2} } sin^nx dx$



#193181 LG

Đã gửi bởi Sk8ter-boi on 03-11-2008 - 00:46 trong Các bài toán Lượng giác khác

Cho tôi hỏi bài diện tích này có ai làm được ko ,post lên cho tôi xem với ,khó quá.
Bài này đẹp nhưng khó
P/s:bài toán hay nhưng ko có ai giải,chán

bài toán trên hay nhưng ko quá khó để tìm ra lời giải bằng lượng giác

sau thời gian khá dài suy nghĩ tui đã có lời giải ko dùng lượng giác .



#187436 *phương trình đặc trưng*

Đã gửi bởi Sk8ter-boi on 28-06-2008 - 10:13 trong Các bài toán Giải tích khác

nếu các hằng số k thỏa mãn đk mà bạn nêu ra thì chúng ta chấp nhận ra nghiệm phức , và th đó nhiều nên mình nghĩ bạn nên đề cập luôn



#186690 P=20cosA+15cosB+12cosC

Đã gửi bởi Sk8ter-boi on 11-06-2008 - 11:15 trong Các bài toán Lượng giác khác

chà , em giải bài trên xài tích vô hướng , làm được tổng quát luôn mà ^^
CM lại cái BDT này $\sum x^2 \geq \sum 2xycosC$ với x;y;z là 3 số thực nào đó
sau đó chỉ cần giải hệ 2xy=a ; 2yz=b ; 2xz=c là ra :(



#186689 phần nguyên

Đã gửi bởi Sk8ter-boi on 11-06-2008 - 11:07 trong Các dạng toán khác

bạn chứng minh được x_n nguyên và chỉ nhận 1 trong 2 giá trị 0 hoặc 1 , sau đó cộng tất cả dãy x_0+x_1+..+x_199 vào đc bao nhiêu thì số đó chính là số các số khác 0



#186501 Bài này rất hay,mọi người vào thử sức

Đã gửi bởi Sk8ter-boi on 06-06-2008 - 22:20 trong Phương trình, hệ phương trình và bất phương trình

xài trò đổi biến x là no của pt trên thì x+1 là no của pt nào ; rồi $\dfrac{1}{x+1}$ là no của cái nào rồi vi-et + 1 tí truy hồi là ra thôi mà



#186500 phần nguyên

Đã gửi bởi Sk8ter-boi on 06-06-2008 - 22:18 trong Các dạng toán khác

bạn nên xem lại đề bài . CHỉ cần với $n \geq 1$ là các số với chỉ số n khác 0



#186499 Logic

Đã gửi bởi Sk8ter-boi on 06-06-2008 - 22:14 trong Các dạng toán khác

thực ra cái việc hình vuông cạnh 1 và đường đi có độ dài m + n thực chất chỉ mang tính thủ tục , chúng ta có thể viết lại như sau :
hình chữ nhật gồm mxn hình vuông , A và B là 2 hình vuông ở 2 đỉnh đối diện , hỏi có bao nhiêu cách đi từ A đến B . Bước đi hợp lệ là bước đi di chuyển giữa 2 ô vuông cạnh chung

gán bước đi sang ngang là 1 ; bước đi dọc là 0 , thì mỗi con đường đi là 1 chuỗi gồm m số 1 và n số 0 . Mà số lượng các chuỗi là $C^n_{m+n}$ , vậy số đường đi là $C^n_{m+n}$



#186236 $\small \sum \dfrac{1}{ h_{a}+1 }= \dfrac{2( p^{2}+...

Đã gửi bởi Sk8ter-boi on 02-06-2008 - 16:32 trong Các bài toán Lượng giác khác

em nghĩ bài này anh nên biến bài chứng minh đẳng thức thành 1 bài dạng khác . Nếu chỉ đơn thuần là chứng minh đẳng thức thì chỉ cần tìm 3 đại lượng Viete của 3 đường cao rồi đổi biến vài lần là ra mà :D , mỗi tội là tính toán dài dòng :lol:



#186097 r=1,R=3,p=5 Tính cosA.cosB.cosC

Đã gửi bởi Sk8ter-boi on 30-05-2008 - 18:42 trong Các bài toán Lượng giác khác

cứ đưa hết về đại số cho lành vậy :leq
cần tính $\prod \dfrac{b^2+c^2-a^2}{2bc}$ , cái này đối xứng nhỉ , nên biểu diễn đc dưới dạng m=a+b+c ; n=ab+bc+ac ; s = abc , mà cả m;n;s đều tính đc
m=2p ; n= p^2+r^2 + 4Rr ; s=4Rrp
tính toán đã đời ...



#186096 Thi tốt nghiệp lớp 12

Đã gửi bởi Sk8ter-boi on 30-05-2008 - 17:59 trong Bất đẳng thức và cực trị

Đề bài này sai rùi, đề phải là
Tìm giá trị lớn nhất và nhỏ nhất của hàm số $f(x)=x+ \dfrac{9}{x} $ trên đoạn [2;4]
Giải:
$f'(x)=1- \dfrac{9}{x^2}= \dfrac{x^2-9}{x^2} $
$f'(x)=0$ và $x \in [2;4]$ :leq x=3 hay x=-3 (loại)
$f(2)= \dfrac{13}{2},f(4)= \dfrac{25}{4} ,f(3)=6 $
Vậy $max f(x)= \dfrac{13}{2}, min f(x)=6$

cho đạo hàm = 0 rồi phải chứng minh rằng đạo hàm cấp cao hơn tại điểm đó khác 0 nữa mới đủ :leq



#186089 Thi tốt nghiệp lớp 12

Đã gửi bởi Sk8ter-boi on 30-05-2008 - 16:46 trong Bất đẳng thức và cực trị

ủa $y=x+ \dfrac{x}{9} = \dfrac{10}{9}x$ sao lại k phải đường thẳng nhỉ :leq



#186086 Thi tốt nghiệp lớp 12

Đã gửi bởi Sk8ter-boi on 30-05-2008 - 16:06 trong Bất đẳng thức và cực trị

anh có nhầm đề không :leq
$y(x)=x+\dfrac{x}{9}$ liên tục , là pt đường thẳng nên nói tóm lại
$y(2) \leq y(x) \leq y(4)$



#186078 Bậc 4

Đã gửi bởi Sk8ter-boi on 30-05-2008 - 13:33 trong Phương trình, hệ phương trình và bất phương trình

hướng giải quyết
giải hệ f'(x)=0 vs f'''(x)=0 ta rút ra trục đối xứng của hàm
thay kết quả vừa thu đc vào . Nói tóm lại ta cần đặt
$x= X-\dfrac{a+b}{2}$ để tiến tới việc giải pt trùng phương



#186042 1 bài thực hành nhỏ

Đã gửi bởi Sk8ter-boi on 29-05-2008 - 21:47 trong Bất đẳng thức và cực trị

chứng minh rằng trong 5 số thực bất kỳ khác nhau thì tồn tại 2 số thỏa mãn BĐT
$|ab+1| > |a-b|$



#186016 2 bài khó và hay

Đã gửi bởi Sk8ter-boi on 29-05-2008 - 16:57 trong Hình học

1)bước 1 : chứng minh tam giác MAB và tam giác MDE đồng dạng góc góc , sử dụng góc đơn thuần
bước 2 gọi đường tròn đường kính MN cắt AB ở điểm thứ 2 K thì từ sim-son và góc nt ta có MDP đồng dạng MAN
nói tóm lại từ 2 điều trên ta có đpcm

2)câu này bạn ghi thiếu vị trí điểm F , F mình nghĩ là giao của DE và BC
a)2 tam giác này đồng dạng góc góc :leq
b)tam giác DGF đồng dạng với tam giác CPH cũng là góc góc nên ta có đpcm :leq
c)do (A,B,F,C)=-1 nên $GF^2 = P(G)/(O) = GQ.GD $nên ta có đpcm
d) gọi (A,Q,D) là (O2) thì $P(G)/(O2) = GQ.GD=GA^2$ và G thuộc ABnên ta có đpcm



#186010 toán rời rạc

Đã gửi bởi Sk8ter-boi on 29-05-2008 - 16:12 trong Các dạng toán khác

bạn có thể giải như sau :leq
1)2n số không vượt quá 2n thì chỉ có n cặp số tự nhiên liên tiếp , mà chúng ta có n+1 số , nên theo nguyên lý chuồng bồ câu ta có đpcm
2) gọi các số đã cho là $\bar{a_i}$
xét dãy {$ \sum\limits_{i=1}^{j} a_i$}_j : j = 1,n dãy trên để lại n số dư khi đem chia cho n .
+)Nếu 1 dãy trên là 1 thặng dư đầy đủ mod n thì rõ ràng ta có đpcm
+)Nếu dãy trên không nhận đủ số dư thì theo nguyên lý chuồng bồ câu , phải tồn tại m và k trong đó $\sum\limits_{i=1}^{m} a_i \equiv \sum\limits_{i=1}^{k} a_$i
Lấy hiệu của chúng ta có đpcm

3)xét 1 dãy n ô trống chứa các ký tự
+)có n-1 cách lấy ra các cắp $a_ia_{i+1}$
+)có 3 cách gán cho mỗi cặp nói trên các ký tự A;B;C
+)có $\bar{C^n_3}$ cách sắp xếp 3 phần tử còn lại vào chỗ
thep quy tắc nhân thì tóm lại có $3(n-1).\bar{C^n_3}$ cách thỏa mãn đề bài

4)trước tiên chú ý điểu sau :
$A_n-A_{n-1} = n$
thật vậy , nếu đã có n-1 đường thẳng trên mặt phẳng , xét đường thứ n . Nó cắt n-1 nói trên ở n-1 điểm nên tạo ra thêm n miền mới
cho n chạy từ 2 đến n rồi cộng lại ta có
$A_n=\dfrac{n(n+1)}{2}+1$



#155653 Thi Hk thui

Đã gửi bởi Sk8ter-boi on 29-04-2007 - 10:53 trong Bất đẳng thức và cực trị

đặt $b+c=x ; a+c=y ; a+b=z$
$\Leftrightarrow \sum \dfrac{z}{x}( \dfrac{y+z-x}{y+z})= \sum_{cyc} \dfrac{z}{x} - \sum \dfrac{z}{y+z} =\sum_{cyc} \dfrac{z}{x} + \sum \dfrac{y}{y+z}\geq \dfrac{9}{2} $
mặt khác RHS $\geq \dfrac{(x+y+z)^2}{xy+yz+zx} + \dfrac{(x+y+z)^2}{xy+yz+zx+x^2+y^2+z^2}= \dfrac{2(x+y+z)^4}{(xy+yz+zx+xy+yz+zx)(xy+yz+zx+x^2+y^2+z^2)} \geq \dfrac{8(x+y+z)^4}{((xy+yz+zx+(x+y+z)^2)^2} \geq \dfrac{9}{2} $



#155572 Đố ai giải được ?....

Đã gửi bởi Sk8ter-boi on 28-04-2007 - 09:54 trong Số học

i think this topic need to be closed urgently !!!!



#155571 Giải giùm em

Đã gửi bởi Sk8ter-boi on 28-04-2007 - 09:49 trong Hình học

1)trước hết $\delta EDA ~ \delta DFC --> AD.DC=EA.CF -->a^2=EA.CF$ ( a là cạnh hthoi)
$--> \delta EAC$ đồng dạng $\delta ACF (cgc) --> \widehat{ACE} = \widehat{AFC} $
$---> \delta AMC$ đồng dạng $ \delta ACF $--> hết phim :)

2) $\delta BHK$ đồng dạng $ \delta ACK$
$-->KH.KA=BK.KC \leq ( \dfrac{BK+KC}{2} )^2=RHS $-->hết phim tập 2



#155550 Toán Quy Nạp

Đã gửi bởi Sk8ter-boi on 27-04-2007 - 22:23 trong Số học

ồ , ko nhất thiết phải cho chứ :) !!
hãy cùng xem :
muốn tính tổng $\sum\limits_{i=1}^{n} i^2$ , tạm thời chưa tìm ra 1 cách biến đổi nào , hãy cùng khảo sát thử vài giá trị
$n=1;2;3;4... $
$ \sum\limits_{i=1}^{n} i=1,3,6,10,15...$
$\sum\limits_{i=1}^{n} i^2 = 1,5,14,30;55...$
với suy nghĩ tự nhiên rằng , 2 tổng trên , bằng 1 cách nào đó có thể biểu diễn dưới đa thức biến n , thiết lập thương
$n=1;2;3;4;5;...$
$\dfrac{\sum\limits_{i=1}^{n} i}{\sum\limits_{i=1}^{n} i^2}=1;\dfrac{5}{3};\dfrac{7}{3};\dfrac{9}{3};\dfrac{11}{3} ...$
từ đây có thể đoán đc công thức rồi , bây giờ mới là nhiệm vụ của quy nạp !!! Khi ta ko xác định đc rõ ràng 1 công thức hay tính chất nào đó , quy nạp là phương pháp rất hữu hiệu vì nó theo đúng 1 suy nghĩ tự nhiên :D